2013 AMC 10B Problems/Problem 13

Revision as of 18:58, 21 February 2013 by Turkeybob777 (talk | contribs) (Created page with "Jo and Blair take turns counting from <math>1</math> to one more than the last number said by the other person. Jo starts by saying "<math>1</math>", so Blair follows by saying "...")
(diff) ← Older revision | Latest revision (diff) | Newer revision → (diff)

Jo and Blair take turns counting from $1$ to one more than the last number said by the other person. Jo starts by saying "$1$", so Blair follows by saying "$1, 2" . Jo then says "$1, 2, 3" , and so on. What is the 53rd number said? $\textbf{(A)}\ 2\qquad\textbf{(B)}\ 3\qquad\textbf{(C)}\ 5\qquad\textbf{(D)}\ 6\qquad\textbf{(E)}\ 8$